LSAT and Law School Admissions Forum

Get expert LSAT preparation and law school admissions advice from PowerScore Test Preparation.

User avatar
 Dave Killoran
PowerScore Staff
  • PowerScore Staff
  • Posts: 5848
  • Joined: Mar 25, 2011
|
#94663
Complete Question Explanation
(The complete setup for this game can be found here: lsat/viewtopic.php?f=280&t=8537)

The correct answer choice is (D)

As established during the setup, at least two French novels must always be selected. Hence, answer choice (D) is correct.
 Melissa_Esposito
  • Posts: 1
  • Joined: Jan 12, 2019
|
#61789
Can you explain how to properly go about solving #10? I selected B but the answer is D and I'm not sure why B is wrong.
 Adam Tyson
PowerScore Staff
  • PowerScore Staff
  • Posts: 5153
  • Joined: Apr 14, 2011
|
#61832
Thanks for the question, Melissa! From the rules, we know that there have to be at least three novels, and at most four novels. We also know that the number of French novels cannot be less than the number of Russian novels. Here's where we will get the answer to this question! Think about the possible combinations of novels, given the third rule about there being at least as many French novels as Russian novels.

Could we have four French novels? No, there are only three.

Could we have three French novels? Sure, no problem. We could also add a fourth novel, a Russian one, and we would be okay with those rules.

Could we have two French novels? Again, sure, and they could be paired with one or two Russian ones and still comply with the rules about novels (at least three, no more than four, and at least as many French ones as Russian ones).

Now, could we have just one French novel? If we did, the other novels (remember, we need at least three of them) would be Russian, either two or three of them to get the number of novels needed, and here's the problem. Now the number of French novels is less than the number of Russian ones! Thus, we must have either two or three French novels in order to comply with the rules, and answer D is what must be true. We have to have at least two French novels.

Think now about answer B, a French novel and a Russian play. Do we have to have both of those? Not at all! In fact, if we try out a hypothetical solution based on the last rule, with both French plays selected, we would not have the Russian play at all. Instead we would have to have two French plays, two French novels, and either one or two Russian novels.

Work through that initial setup again and see if there are any other inferences you can make. Must we, for example, have any Russian novels in our solution? Give that a try, and see if there are any other inferences you can come up with.

Get the most out of your LSAT Prep Plus subscription.

Analyze and track your performance with our Testing and Analytics Package.